Difference between revisions of "2006 AMC 12A Problems/Problem 21"

Line 1: Line 1:
 
== Problem ==
 
== Problem ==
 +
 +
Let
 +
 +
<math>S_1=\{(x,y)|\log_{10}(1+x^2+y^2)\le 1+\log_{10}(x+y)\}</math>
 +
 +
and
 +
 +
<math>S_2=\{(x,y)|\log_{10}(2+x^2+y^2)\le 2+\log_{10}(x+y)\}</math>.
 +
 +
What is the ratio of the area of <math>S_2</math> to the area of <math>S_1</math>?
 +
 +
<math> \mathrm{(A) \ } 98\qquad \mathrm{(B) \ } 99\qquad \mathrm{(C) \ } 100\qquad \mathrm{(D) \ } 101\qquad \mathrm{(E) \ }  102</math>
  
 
== Solution ==
 
== Solution ==

Revision as of 00:07, 11 July 2006

Problem

Let

$S_1=\{(x,y)|\log_{10}(1+x^2+y^2)\le 1+\log_{10}(x+y)\}$

and

$S_2=\{(x,y)|\log_{10}(2+x^2+y^2)\le 2+\log_{10}(x+y)\}$.

What is the ratio of the area of $S_2$ to the area of $S_1$?

$\mathrm{(A) \ } 98\qquad \mathrm{(B) \ } 99\qquad \mathrm{(C) \ } 100\qquad \mathrm{(D) \ } 101\qquad \mathrm{(E) \ }  102$

Solution

See also